Particles and Fields – a neverending story

In summary: The point is, how is the state defined mathematically in QFT which is represented by the matrix element above? Is it defined at all?In summary, the conversation revolves around the issue of particle localization and the existence of a position operator in QFT. The speaker is attempting to understand the connection between QM and QFT, specifically in relation to the single-particle wave function and its transformation behavior. However, there is no generally accepted answer to this question and there are some disagreements and misunderstandings about the nature of particle states and their properties in QFT. Some sources suggest that there is no position operator and no states in position space in QFT, while others argue that a particle can be localized in QFT, but with
  • #1
ledamage
36
0
It is often said that the field operator [tex]\hat{\phi}(x)[/tex] of some free field theory, e.g. Klein-Gordon or Dirac, acting on the vacuum state, creates "one particle localized at x" or "in a position eigenstate at x".

For example, from the Fourier expansion of the real Klein-Gordon field operator, we get

[tex]\hat{\phi}(x) |0\rangle = \int \mathrm{d}^3 \tilde{p} \: f_p^*(x) a^\dagger(p) |0\rangle \ , [/tex]

where [tex]\mathrm{d}^3 \tilde{p} \equiv \mathrm{d}^3 p/2p^0[/tex] is the Lorentz covariant measure and [tex]f_p(x)[/tex] are the plane wave solutions to the Klein-Gordon equation.

Now, this state could be regarded as a localized one-particle state if [tex]f_p^*(x)[/tex] is the ordinary QM position space wave-function of the particle in momentum eigenstate [tex]|\mathbf{p}\rangle[/tex], namely [tex]f_p^*(x)=\langle \mathbf{p} |\mathbf{x} \rangle[/tex]. Writing [tex]a^\dagger(p) |0\rangle \equiv |\mathbf{p}\rangle[/tex], the above equation would turn into

[tex]\hat{\phi}(x) |0\rangle = \int \mathrm{d}^3 \tilde{p} \: |\mathbf{p} \rangle \langle \mathbf{p} |\mathbf{x} \rangle \ ,[/tex]

which would somehow be related to [tex]|\mathbf{x}\rangle[/tex] since the first part resembles an identity operator. But two problems occur:

1. What is [tex]|\mathbf{x}\rangle[/tex]? While I can define the state of "one particle with momentum p" to be [tex]a^\dagger(p) |0\rangle \equiv |\mathbf{p}\rangle[/tex], I have no idea how a state like [tex]|\mathbf{x}\rangle[/tex] can canonically arise in a QFT. My only idea is to promote the equation

[tex]f_p^*(x)=\langle \mathbf{p} |\mathbf{x} \rangle[/tex]

to a defining axiom for [tex]|\mathbf{x}\rangle[/tex], that is, postulating that the free field solutions to the real Klein Gordon equation can be interpreted as one-particle wave functions in the ordinary QM sense. But this would mean you have to add a new axiom relating QM and QFT?!

2. I have some doubts that I can regard [tex]\int \mathrm{d}^3 \tilde{p} \: |\mathbf{p} \rangle \langle \mathbf{p} |[/tex] as an identity operator because of the covariant measure.

Can anyone help me? Thanks!
 
Last edited:
Physics news on Phys.org
  • #2
The problem with the states
[tex]|\mathbf{x}\rangle = \hat{\phi}(\mathbf{x},0)|0\rangle[/tex]
is that they are not orthogonal.
Namely,
[tex]\langle\mathbf{x}'|\mathbf{x}\rangle \neq \delta^3(\mathbf{x}-\mathbf{x}')[/tex]
Note, however, that they become orthogonal in the non-relativistic limit.
 
  • #3
Thanks for the reply! I a book about group theory, the basic relation

[tex]\psi(x) = \langle 0 | \hat{\psi}(x)| \mathbf{p}\rangle[/tex]

is used to obtain the transformation behavior of the field operator from the transformation behavior of the single-particle wave function. In a way, I use the representation theory of the Poincaré group to construct QM single-particle basis states and relate their transformation behavior to field operators of QFT by the above equation. Is this right?

But I don't get the connection; I think this is a subtle point: from the viewpoint of second quantization, everything is clear because one explicitly constructs the possibility to create and annihilate particles, so the above equation just follows mathematically.

But from the viewpoint of field quantization, I never introduced single-particle wave functions, I just applied the QM axioms to fields so that there is no obvious interpretation of the function which is represented by the above matrix element.
Does this make any sense?
 
Last edited:
  • #5
Wow, this is odd! I have found this nowhere in standard textbooks on that topic...instead I spent hours blaming myself for not getting this allegedly obvious relation...

Thanks very much!
 
  • #6
There is no position operator and no states in position space in QFT!

Read the first 7 pages of these https://www.physicsforums.com/newreply.php?do=newreply&noquote=1&p=1784740" .
 
Last edited by a moderator:
  • #7
Icosahedron said:
There is no position operator and no states in position space in QFT!

Read the first 7 pages of these https://www.physicsforums.com/newreply.php?do=newreply&noquote=1&p=1784740" .
It seems that you gave a wrong link. Can you correct it?
 
Last edited by a moderator:
  • #8
http://www2.physics.utoronto.ca/~luke/PHY2403/References_files/lecturenotes.pdf

Otherwise, Srednicki writes in his book on page 10, that keeping X and promoting t to an operator can also be done to get a relativistic particle theory.

Which leaves me puzzled.
 
Last edited by a moderator:
  • #9
Icosahedron said:
http://www2.physics.utoronto.ca/~luke/PHY2403/References_files/lecturenotes.pdf
It contains some frequent wrong claims.

For example, it says that a particle cannot be localized in a region smaller than its Compton length. However, if it was true, it would imply that a massless photon could not be localized in any region smaller than its Compton length. On the other hand, the position of the photon can be measured by certain accuracy.

Next, it says that in relativistic QM there is no such thing as a 2, 1, or 0 particle state. But for free relativistic fields, such things certainly do exist. Moreover, a 1 and 0 particle states are stable even for interacting particles.
 
Last edited by a moderator:
  • #10
On the other hand, the position of the photon can be measured by certain accuracy.

Position of a photon? But you know that a photon has no rest frame...

Next, it says that in relativistic QM there is no such thing as a 2, 1, or 0 particle state.

Right. I think every QFT text starts with this observation. What is wrong with it?
 
  • #11
Icosahedron said:
1. Position of a photon? But you know that a photon has no rest frame...

2. I think every QFT text starts with this observation. What is wrong with it?
1. So what? Take for example a classical photon. It has no rest frame too. Yet, at any time it has a definite position. The position, of course, changes with time, but it does not mean that the position does not exist.

2. For example, what is the state |0>, if not the state with exactly zero number of particles? And no, not every QFT text starts with this observation.
 
Last edited:
  • #12
Demystifier said:
For example, it says that a particle cannot be localized in a region smaller than its Compton length.

This is a point related to my question. What does particle localization mean in QFT? I am able to superpose some one-particle momentum eigenstates. But to determine the position of the "quantum object" thus obtained I need a position operator which does not exist. I cannot see more than a formal analogy to second quantization where it is just a mathematical implication that the Fourier transform of the superposition weight function is the one-particle wave function where an ordinary QM position operator can act on.
 
  • #13
ledamage said:
What does particle localization mean in QFT?

It depends first on how you define "particle". I.e., which group you start with for which
you find the unitary irreducible representations. The Poincare algebra doesn't come
with a position generator, so some people advocate constructing one (Newton-Wigner)
from the enveloping ring. Other people are unimpressed with such "schtick". :-)

The x parameters in orthodox QFT (and Minkowski space in general) just serve as
a representation space for the Poincare group. The vacuum state has deterministic
energy-momentum of 0, and therefore cannot be a localised state in the usual sense.
Some people have suggested that this fact is at the core of the Reeh-Schlieder
paradox (which "proves" that fields over "there" can be recovered entirely from
fields "here" -- where "here" and "there" mean spacelike-separated regions).

Here's some fairly recent papers that debate these still-controversial points.

1) Fleming, "Reeh-Schlieder Meets Newton-Wigner", available via
http://philsci-archive.pitt.edu/archive/00000649/00/RS_meets_NW,_PDF.pdf

Abstract:
The Reeh-Schlieder theorem asserts the vacuum and certain other states to be spacelike
superentangled relative to local fields. This motivates an inquiry into the physical status
of various concepts of localization. It is argued that a covariant generalization of Newton-
Wigner localization is a physically illuminating concept. When analyzed in terms of
nonlocally covariant quantum fields, creating and annihilating quanta in Newton-Wigner
localized states, the vacuum is seen to not possesses the spacelike superentanglement that
the Reeh-Schlieder theorem displays relative to local fields, and to be locally empty as
well as globally empty. Newton-Wigner localization is then shown to be physically
interpretable in terms of a covariant generalization of the center of energy, the two
localizations being identical if the system has no internal angular momentum. Finally,
some of the counterintuitive features of Newton-Wigner localization are shown to have
close analogues in classical special relativity.


2) Halvorson, "Reeh-Schlieder defeats Newton-Wigner: On alternative localization
schemes in relativistic quantum field theory".
Available as quant-ph/0007060.

Abstract:
Many of the “counterintuitive” features of relativistic quantum
field theory have their formal root in the Reeh-Schlieder theorem,
which in particular entails that local operations applied to the vacuum
state can produce any state of the entire field. It is of great interest
then that I.E. Segal and, more recently, G. Fleming (in a paper entitled
“Reeh-Schlieder meets Newton-Wigner”) have proposed an alternative
“Newton-Wigner” localization scheme that avoids the Reeh-Schlieder
theorem. In this paper, I reconstruct the Newton-Wigner localization
scheme and clarify the limited extent to which it avoids the counterin-
tuitive consequences of the Reeh-Schlieder theorem. I also argue that
there is no coherent interpretation of the Newton-Wigner localization
scheme that renders it free from act-outcome correlations at spacelike
separation.


3) DeBievre, "Where's that quantum"? Available as math-ph/0607044

Abstract:
The nature and properties of the vacuum as well as the meaning
and localization properties of one or many particle states have at-
tracted a fair amount of attention and stirred up sometimes heated
debate in relativistic quantum field theory over the years. I will review
some of the literature on the subject and will then show that these is-
sues arise just as well in non-relativistic theories of extended systems,
such as free bose fields. I will argue they should as such not have given
rise either to surprise or to controversy. They are in fact the result
of the misinterpretation of the vacuum as “empty space” and of a too
stringent interpretation of field quanta as point particles. I will in par-
ticular present a generalization of an apparently little known theorem
of Knight on the non-localizability of field quanta, Licht’s character-
ization of localized excitations of the vacuum, and explain how the
physical consequences of the Reeh-Schlieder theorem on the cyclic-
ity and separability of the vacuum for local observables are already
perfectly familiar from non-relativistic systems of coupled oscillators.
 
Last edited by a moderator:
  • #14
Thanks very much for the detailed reply! I will have a look at some of these papers. But I'm still confused why in standard textbooks on QFT, say, a Gaussian superposition of momentum eigenstates is called localized (blurred around a certain point x) in space. Which reasoning is behind this?
 
  • #15
ledamage said:
[...] But I'm still confused why in standard textbooks on QFT,
Did you have specific textbook quote in mind?

say, a Gaussian superposition of momentum eigenstates is called localized
(blurred around a certain point x) in space. Which reasoning is behind this?

IMHO, I think that's just what people would like to think. I.e., we'd like to
think that the x's we use when constructing orthodox QFT do indeed correspond
meaningfully to points in physical space. But one must delve into the subtleties
of position operators, understand the Reeh-Schlieder theorem, etc, (and maybe
also Haag's theorem), before it becomes apparent that something is deeply
puzzling about this whole subject.

Of course, such delving is clearly not necessary to calculate the stunningly
accurate predictions of QFT, hence most people don't worry very much about it.
 
  • #16
strangerep said:
Did you have specific textbook quote in mind?

Srednicki for example, Chapter 5 (LSZ formula), Eqs. (5.6) and (5.7). He speaks of creating a particle localized in momentum and position space. (A draft of the book is available at http://www.physics.ucsb.edu/~mark/qft.html" .)

Okay, thanks to you all for your answers! I think I know now what to read to get on...
 
Last edited by a moderator:
  • #17
Strangerep, thanks for posting those references. I'll definitely take a closer look at the last one and maybe the other two as well some time soon.

You seem to know this stuff pretty well already, so I'd like to ask you specifically about something that I have believed to be true until now. (Right now I'm really confused. I have e.g. never heard of the Reeh-Schlieder theorem before). I've been saying things like this:
Fredrik said:
The state of a photon is in general a superposition of states with different momenta. Let's ignore other degrees of freedom and express this as

[tex]\int d^3p f(\vec p)a^\dagger(\vec p)|0\rangle[/tex]

where [itex]a^\dagger(\vec p)[/itex] is the creation operator that creates a one-particle state with momentum p when it acts on the vaccum. The Fourier transform of f can be interpreted as an ordinary wave function.
Is this completely false? I guess I always thought that since the f above is pretty much the same in relativistic QM as in non-relativistic QM, its Fourier transform should also be pretty much the same as in non-relativistic QM.
 
  • #18
Fredrik said:
Is this completely false? I guess I always thought that since the f above is pretty much the same in relativistic QM as in non-relativistic QM, its Fourier transform should also be pretty much the same as in non-relativistic QM.
The right questions are: Can this Fourier transform be interpreted as the probability amplitude? In other words, is such a probability conserved? And is it defined in a relativistic covariant way?
See e.g. the Appendix in
http://xxx.lanl.gov/abs/0804.4564 [not yet published]
 
  • #19
Demystifier said:
The right questions are: Can this Fourier transform be interpreted as the probability amplitude?

If I get this right, this is the same as asking whether you can interpret a first-quantized field as a particle system obtained by second quantization which obviously is done without ever asking for reasons.
 
  • #20
Fredrik said:
Strangerep, thanks for posting those references. I'll definitely take a closer look at the last one and maybe the other two as well some time soon.
You seem to know this stuff pretty well already, so [...]
(Urgle!?) No, I'm not an expert. I'm aware that the various puzzles exist, but I
don't know how to resolve them satisfactorily. (Actually, I don't think
anyone does - hence the ongoing debate.)
 
  • #21
ledamage said:
If I get this right, this is the same as asking whether you can interpret a first-quantized field as a particle system obtained by second quantization which obviously is done without ever asking for reasons.
No, that's not the question here. The [itex]a^\dagger(\vec p)|0\rangle[/itex] states are eigenstates of the momentum operator that you can construct from the quantum field. (Noether's theorem tells us how). The question is about the meaning of the Fourier transform of the coefficients when an arbitrary state is expressed as a "linear combination" of momentum eigenstates.

In non-relativistic QM, that Fourier transform is the wave function, and as you know the square of its absolute value is a probability density describing where in space the particle is likely to be found. I've been assuming that the interpretation as a probability density holds in relativistic QM too, but I'm still not sure how correct that is. (I still haven't had time to really think about Demystifier's answer. It looks like a very good one but I need to think some more about what it means).
 
  • #22
Fredrik said:
No, that's not the question here. The [itex]a^\dagger(\vec p)|0\rangle[/itex] states are eigenstates of the momentum operator that you can construct from the quantum field. (Noether's theorem tells us how). The question is about the meaning of the Fourier transform of the coefficients when an arbitrary state is expressed as a "linear combination" of momentum eigenstates.

But this is exactly what I meant, isn't it? You treat a first-quantized field like a second quantized theory by interpreting

[tex]\psi(x_1, \hdots, x_n) := \langle 0 | \hat{\psi}(x_1) \cdots \hat{\psi}(x_n) | \alpha \rangle[/tex]

as a many-particle wave function, where [tex]|\alpha\rangle[/tex] is some superposition of momentum eigenstates. But the point is that the matrix element [tex]\langle 0 | \hat{\psi}(x_1) \cdots \hat{\psi}(x_n) | \alpha \rangle[/tex] itself gives no key to this interpretation since the field operator is obtained by first quantization. And in fact, as has been pointed out here, there are reasonable doubts to do so.
 
  • #23
ledamage said:
You treat a first-quantized field like a second quantized theory by interpreting

[tex]\psi(x_1, \hdots, x_n) := \langle 0 | \hat{\psi}(x_1) \cdots \hat{\psi}(x_n) | \alpha \rangle[/tex]

as a many-particle wave function, where [tex]|\alpha\rangle[/tex] is some superposition of momentum eigenstates. [...]

In such areas, it helps to go back and think about how the multi-particle Fock space
is constructed. One starts with a (single-particle) Hilbert space [itex]H_1[/itex] carrying
a +ve energy representation of the Poincare group. By various abstract nonsense
(the spectral theorem from Functional Analysis) we can then assert that [itex]H_1[/itex]
is spanned by momentum eigenstates, subject to the mass-shell condition
[itex]\delta^{(4)}(p^2 - m^2)[/itex].

If we try to Fourier-transform to position space (i.e., to obtain an alternate basis
of position eigenstates), it ceases to be physically meaningful if we don't include
the [itex]\delta^{(4)}(p^2 - m^2)[/itex] factor in the Fourier transform.
Remember now that a product in momentum space Fourier-transforms into a
convolution in position space, so you don't get very nice position eigenstates.
(IMHO, this is another reason why we hit paradoxes like Reeh-Schlieder.)

Now, what is a 2-particle Hilbert space? We take the tensor product of two
copies of [itex]H_1[/itex], i.e., [itex]H_2 := H_1 \otimes H_1[/itex].
Remember: I'm still thinking of these in momentum basis, or rather, two tensored
copies of momentum space, each with their separate [itex]\delta^{(4)}(p^2 - m^2)[/itex]
constraints. Now think about the horrors when you try to Fourier-transform
this tensor product to a 2-position basis. Convolutions everywhere. (And even
that's only if you can somehow make the Fourier transform itself make sense.)

And yet, we try to kid ourselves that the tensor product of 2 separately-transformed
[itex]H_1[/itex] spaces can be handled in the simple way shown at the start of
this post. Really, it becomes mathematical nonsense very quickly. You can't
work with infinite-dimensional spaces and their duals sensibly without
paying careful attention to topological matters, unboundedness of operators,
and the like.

(That probably didn't help, I know. :-)
 
  • #24
If you assume that the position and momentum space wave functions [tex]\psi(x)[/tex] and [tex]\hat{\psi}(p)[/tex] are related by the Fourier transforms, define the single particle state as

[tex]
|\psi\rangle = \int d^3p\; \hat{\psi}(p) a_p^{\dagger}|0\rangle,
[/tex]

derive the time evolution

[tex]
|\psi(t)\rangle = \int d^3p\; \hat{\psi}(p) e^{-iE_p t} a_p^{\dagger}|0\rangle
[/tex]

from the Shrödinger's equation (after fixing the Hamilton's operator from whatever QFT principles you want to postulate initially...) (here [tex]E_p=\sqrt{|p|^2+m^2}[/tex]) and then recognize the evolving momentum space wave function

[tex]
\hat{\psi}(t,p) = \hat{\psi}(p) e^{-iE_p t},
[/tex]

it follows that in the position space representation the time evolution is given by the old-fashioned relativistic Schrödinger's equation

[tex]
\psi(t,x) = e^{-it\sqrt{-\nabla^2 + m^2}}\psi(x),
[/tex]

which satisfies the normalization conserving condition

[tex]
\int d^3x\; |\psi(t,x)|^2 = \int d^3x\; |\psi(x)|^2 = 1
[/tex]

at all times. To me this looks all good, and I must say I'm slightly confused about why this (assuming I've understood the mainstream view correctly) is widely considered incorrect. The QFT books always start doing some tricks with the [tex]1/(2E_p)[/tex] factor, and that's where I usually get lost.
 
  • #25
jostpuur said:
[...] define the single particle state as

[tex]
|\psi\rangle = \int d^3p\; \hat{\psi}(p) a_p^{\dagger}|0\rangle,
[/tex]

Aren't you in trouble already, even at this early stage? You don't
have the [itex]E_p[/itex] factor in the denominator, so maybe
this expression is not Lorentz-covariant?
 
  • #26
strangerep said:
Aren't you in trouble already, even at this early stage? You don't
have the [itex]E_p[/itex] factor in the denominator, so maybe
this expression is not Lorentz-covariant?

Hello. Immediately after posting the previous post, I realized there was several places where I could have still placed energy-factors so that I would have still understood what I'm doing, but I decided not to try any quick edit fixes. About your question... I'm not sure if that's trouble. It's more like a convention question. I think there are several equivalent ways to do this stuff, and the stuff gets confusing because it is usually not clear what conventions are being used.

For example. Suppose I had defined [tex]|p\rangle = \sqrt{2E_p} a_p^{\dagger}|0\rangle[/tex]. Then I could have defined the state like

[tex]
|\psi\rangle = \int \frac{d^3p}{\sqrt{2E_p}} \hat{\psi}(p)|p\rangle,
[/tex]

and actually nothing changed! Of course I could also try to define the state like

[tex]
\int d^3p\; \hat{\psi}(p)|p\rangle
[/tex]

or

[tex]
\int \frac{d^3p}{\sqrt{2E_p}} \hat{\psi}(p)a_p^{\dagger}|0\rangle
[/tex]

and these expressions would not be equivalent.

Besides the raising operator, different factors could also be absorbed into the definition of the wave function [tex]\hat{\psi}[/tex] too, but I better not say anything more now, because I'm not fully sure what all these conventions would mean right now.
 
Last edited:
  • #27
jostpuur said:
[...] I realized there was several places where I could have still placed energy-factors so that I would have still understood what I'm doing, but I decided not to try any quick edit fixes. About your question... I'm not sure if that's trouble. It's more like a convention question. I think there are several equivalent ways to do this stuff, and the stuff gets confusing because it is usually not clear what conventions are being used.
The crucial part is to be clear first about what should be
Lorentz invariant/covariant under any convention.

E.g., P&S use a Lorentz-invariant normalization -- see their eq 2.36, i.e.,

[tex]
\langle p|q\rangle ~=~ 2E_p \, (2\pi)^3 \, \delta^{(3)}(p-q) ~,
[/tex]

(which corresponds to your earlier [itex]|p\rangle = \sqrt{2E_p} a_p^{\dagger}|0\rangle[/itex]),
and the discussion surrounding it about how to make a 3D delta fn invariant.
 
  • #28
Jostpuur, strangerep is right that your expressions are not Lorentz invariant. See also the Appendix in my paper with correct Lorentz-invariant measures.
 
  • #29
(edit: Ouch. Using \vec{p}' wasn't so good idea, because the prime gets messed into the arrow. Try to manage with it...)

My point in the post #26 was that the mere appearance of the energy-factor doesn't tell if things are going right or wrong, because that factor can be made appear or disappear with change of conventions. The post wasn't fully logical though, but a slightly heuristic instead.

I must admit that I'm not fully sure about the meaning of "covariant". I've understood that it is related to some form of expression remaining the same in some situations, but what's the difference between calling the expression invariant? Anyway, I'll have a careful and critical look at these integrals here:

Using on-shell four momentums it is possible to define [tex]\Lambda:\mathbb{R}^3\to\mathbb{R}^3[/tex] that transforms three momentum [tex]\Lambda(\vec{p})=\vec{p}'[/tex]. With a boost in 3-direction we have

[tex]
p^3 = \frac{(p')^3 + uE_{\vec{p}'}}{\sqrt{1-u^2}}
[/tex]

[tex]
\frac{dp^3}{d(p')^3} = \frac{1 + u(p')^3/E_{\vec{p}'}}{\sqrt{1-u^2}} = \frac{1}{E_{\vec{p}'}}\frac{E_{\vec{p}'} + u(p')^3}{\sqrt{1-u^2}} = \frac{E_{\vec{p}}}{E_{\vec{p}'}}
[/tex]

Some arguments of rotational symmetry reveal that with arbitrary boost we have

[tex]
\textrm{det}(D\Lambda^{-1}(\vec{p}')) = \frac{E_{\vec{p}(\vec{p}')}}{E_{\vec{p}'}},\quad \vec{p}(\vec{p}') =\Lambda^{-1}(\vec{p}').
[/tex]

This means, that if we have some function [tex]f:\mathbb{R}^3\to\mathbb{R}[/tex], and an integral

[tex]
\int d^3p\; f(\vec{p}),
[/tex]

we can perform a change of variables, and write the same integral as an integral over variable [tex]\vec{p}'[/tex]. The integral is then

[tex]
\int d^3p'\;\textrm{det}(D\Lambda^{-1}(\vec{p}')) f(\vec{p}') = \int d^3p'\; \frac{E_{\vec{p}(\vec{p}')}}{E_{\vec{p}'}} f(\vec{p}').
[/tex]

This shows that on the other hand an expression

[tex]
\int \frac{d^3p}{E_{\vec{p}}} f(\vec{p})
[/tex]

has the particular covariant form, because under the change of variables it becomes

[tex]
\int \frac{d^3p'}{E_{\vec{p}'}} f(\vec{p}').
[/tex]

So far this is easy, because it is merely calculating. Things get more difficult when you say that the equation

[tex]
|\psi\rangle = \int d^3p\;\hat{\psi}(\vec{p})a_{\vec{p}}^{\dagger}|0\rangle
[/tex]

is not properly covariant or invariant. Do you know what you meant by this yourselves? We are never going to be interested in the change of variable [tex]\vec{p}'=\Lambda(\vec{p})[/tex] in this integral. It would have no meaning. If you have a wave function [tex]\hat{\psi}(t,\vec{p})[/tex] in the space time, then it is possible to solve a boosted wave function [tex]\hat{\psi}'(t',\vec{p}')=\hat{\psi}(t,\vec{p})[/tex], but there does not exist any boost for a wave function defined at some instant. For this reason, there does not exist boost for the state vector. There is no transformation [tex]|\psi\rangle\mapsto \Lambda|\psi\rangle[/tex]. Because there does not exist Lorentz boosts for the left or the right side of the equation, it does not make sense to ask if the equation is covariant or invariant. All there exists is one change of variable, which in this context has no clear meaning. The integrals where this change of variable is related to boosts occur in different kind of situations, and I believe that in those the energy-factor plays important role.

My point in the post #24 was that the way how this convention gives the conserving probabilities looks very good. Because of this, it is difficult for me to believe that it could be simply wrong. Instead, if there exists some other expression to write the single particle state, which for sure is right, my first thought is that these expressions must be equivalent in some sense then.
 
Last edited:
  • #30
Jostpuur, you have not specified the commutation relations among the creation and destruction operators, so it is impossible to say whether your alternative representations of the same thing are all invariant or all non-invariant. See the second equation of my Appendix (Eq. (44) in v2 of arXiv:0804.4564).

But this is not the real problem. Even if you have fixed these normalizations such that the wave function is invariant, the problematic expression is
[tex] \int d^3x |\psi({\bf x},t)|^2 [/tex]
This expression is not Lorentz invariant. Compare with Eq. (49) in v2 of arXiv:0804.4564, which is Lorentz invariant.
 
  • #31
jostpuur said:
I must admit that I'm not fully sure about the meaning of "covariant". I've understood that it is related to some form of expression remaining the same in some situations,
That's "invariant".

"Lorentz-covariant expression" means that the expression transforms as a tensor under
Lorentz transformations. The crucially important feature about this is that if a covariant
expression is 0 in one frame of reference, it is 0 in every frame of reference.
(That's the mathematical representation of physical statements like "the laws of physics are
the same for all inertial observers".)

[...] Things get more difficult when you say that the equation

[tex]
|\psi\rangle = \int d^3p\;\hat{\psi}(\vec{p})a_{\vec{p}}^{\dagger}|0\rangle
[/tex]

is not properly covariant or invariant. Do you know what you meant by this yourselves?
Well, I'm pretty sure I know what I meant. :-) And I'd bet a large amount of money
that Demystifier does too.

Here, one is dealing with Fock space (I'll assume bosonic, so that canonical commutation
relations apply to the annihilation/creation operators). It is an essential part of the original
construction of any such Fock space that it should "carry" an irreducible representation of
the full Poincare group. In less high-falutin' language, this means that (eg), given an abstract
Lorentz transformation [itex]\Lambda[/itex] (eg a boost in some direction), we can find
a concrete representation [itex]U_\Lambda[/itex], (constructed somehow from the basic
annihilation and creation operators), such that
[itex]U_\lambda |\psi\rangle = |\Lambda\,\psi\rangle[/itex], and such that we can
satisfy the various commutation relations of the Poincare group. (The "irreducible" adjective
earlier means here that any operator on the Fock space can be constructed from
the basic annihilation and creation operators.)

Further, under a Lorentz boost that takes [itex]\vec{p} \to \vec{p'} := \Lambda \vec{p} [/itex],
we have

[tex]
a_{\vec{p'}} ~=~ U_\Lambda ~ a_{\vec{p}}^{\dagger} ~ U_\Lambda^\dagger
~=~ a_{\Lambda \vec{p}}^{\dagger}
[/tex]

We are never going to be interested in the change of variable [tex]\vec{p}'=\Lambda(\vec{p})[/tex] in this integral.It would have no meaning.
Sure we might, sure it does...

We could perform a rotation, or we could transform between the lab frame
and the center-of-momentum frame, or heaps of other things to make our life easier when
calculating stuff. We could also transform between Cartesian [itex]p_1, p_2, p_3[/itex] to
polar [itex]p_r, p_\theta, p_\phi[/itex] (which sometimes makes integrals easier to solve.

For this reason, there does not exist boost for the state vector. There is no transformation [tex]|\psi\rangle\mapsto \Lambda|\psi\rangle[/tex]. Because there does not exist Lorentz boosts for the left or the right side of the equation,
Yes, there does. Yes, there is. Yes, there are.

it does not make sense to ask if the equation is covariant or invariant.
And yes, it does.

All there exists is one change of variable, which in this context has no clear meaning.
The equation

[tex]
|\psi\rangle = \int d^3p\;\hat{\psi}(\vec{p})a_{\vec{p}}^{\dagger}|0\rangle
[/tex]

expresses that the state vector [itex]|\psi\rangle[/itex] is a linear combination of
the basis state vectors [itex]a_{\vec{p}}^{\dagger}|0\rangle[/itex]. Under the
Lorentz transform [itex]U_\Lambda[/itex], we have:

[tex]
U_\Lambda |\psi\rangle
~=~ U_\Lambda \int d^3p\;\hat{\psi}(\vec{p})a_{\vec{p}}^{\dagger}|0\rangle
~=~ U_\Lambda \int d^3p\;\hat{\psi}(\vec{p})a_{\vec{p}}^{\dagger}
U_\Lambda^{\dagger} U_\Lambda |0\rangle
~=~ U_\Lambda \int d^3p\;\hat{\psi}(\vec{p})a_{\vec{p}}^{\dagger}
U_\Lambda^{\dagger} |0\rangle
[/tex]

where I have used the assumption that [itex]U_\Lambda |0\rangle = |0\rangle[/itex],
i.e., that the vacuum is Lorentz-invariant.

[continued in next post...]
 
Last edited:
  • #32
Continuing my previous too-long post...

I wrote
[tex]
U_\Lambda |\psi\rangle
~=~ U_\Lambda \int d^3p\;\hat{\psi}(\vec{p})a_{\vec{p}}^{\dagger}|0\rangle
~=~ U_\Lambda \int d^3p\;\hat{\psi}(\vec{p})a_{\vec{p}}^{\dagger}
U_\Lambda^{\dagger} U_\Lambda |0\rangle
~=~ U_\Lambda \int d^3p\;\hat{\psi}(\vec{p})a_{\vec{p}}^{\dagger}
U_\Lambda^{\dagger} |0\rangle
[/tex]

where I have used the assumption that [itex]U_\Lambda |0\rangle = |0\rangle[/itex],
i.e., that the vacuum is Lorentz-invariant.

Now, the [itex]U_\Lambda[/itex] can only be moved legally through to
integration operation to act on the integrand if the measure [itex]d^3p[/itex]
is Lorentz-invariant. But it's not, so we need to go back and start instead with the different
Lorentz-invariant measure [itex]d^3p/E_p[/itex]. In that case, we could move
the [itex]U_\Lambda[/itex] through to the left side of [itex]a_{\vec{p}}^{\dagger}[/itex]
and get the original state vector, re-expressed in terms of p' basis instead of the
original p basis. (Yes, I know I should probably make that clearer, but I need to go.)

Back to Jostpuur's
My point in the post #24 was that the way how this convention gives the
conserving probabilities looks very good. Because of this, it is difficult for
me to believe that it could be simply wrong.
The tricky parts happen when one tries to combine the probability concepts
sensibly with special relativity.
 
  • #33
Quantum mechanics uses partial differential equations to model how probability densities evolve over time. This produces `waves' of probability densities not unlike the waves on the surface of the ocean. The higher the wave at a given point the more likely the particle is to be located at that point. Everything that exists has a wave aspect. One cannot model what the particle does between observations of the particle. One can only model how the probability of observing it changes over time. These probabilities have a wave like character. Everything also has a particle aspect. Every particle interacts with other particles as if it were located at a single point in space.
It is these two aspects of a particle that causes physicists to speak of the ``collapse'' of the wave function. Prior to observing a particle it could be located at anywhere the wave function is not zero. After observing it the particle is known to be within a much smaller region of space. The wave function has collapsed. Of course there cannot be a physical wave function that collapses because this would violate special relativity. However the wave function does at times seem to be a physical entity because wave functions from two particles can interfere with each other just as physical waves do.
 
  • #34
This debate keeps popping up on these boards, and I'll side with Strangereps point of view on the subject.

I thought i'd add a classic thought experiment about localization often taught in early QFT courses (eg Coleman's lectures). This very much relates to the nonvanishing of the propagator outside lightcone and things like that, as well as the early tensions between field theory and relativistic QM.

If you take a one particle state in a box (defined at the moment, purely in terms of relativistic qm, not field theory), and you squeeze it past the compton wavelength of the particle. That is exactly the regime where pair creation processes start becoming kinematically important on purely dimensional analysis grounds. So the more you squeeze, the more the occupancy number becomes a bad 'state', and you are forced to no longer deal with 1 particle, but rather many. You've essentially lost localization and your would be position operator becomes mathematically illdefined.

Enter field theory, which is a theory with infinite degrees of freedom, and the (somewhat sad) requirement to only use momentum operators.

You are free to try to sidestep this somehow (see Newton Wigner) but it starts becoming more and more intractable as you add interaction terms, and the technical and conceptual difficulties start increasing (the photon for instance becomes the source of much debate).
 
  • #35
strangerep said:
Continuing my previous too-long post...

I wrote ?Now, the [itex]U_\Lambda[/itex] can only be moved legally through to
integration operation to act on the integrand if the measure [itex]d^3p[/itex]
is Lorentz-invariant. But it's not, so we need to go back and start instead with the different
Lorentz-invariant measure [itex]d^3p/E_p[/itex]. In that case, we could move
the [itex]U_\Lambda[/itex] through to the left side of [itex]a_{\vec{p}}^{\dagger}[/itex]
and get the original state vector, re-expressed in terms of p' basis instead of the
original p basis. (Yes, I know I should probably make that clearer, but I need to go.)

Back to Jostpuur's

The tricky parts happen when one tries to combine the probability concepts
sensibly with special relativity.

In my opinion, strangerep and demystifier have written excellent posts in this thread. One small quibble, however.

Normally the unitary transformation operator, U, will act on the "a" operators and the vacuum state. U will commute with p and the volume measure in p-space. Thus you'll have something like UaU*, which will give the Lorentz transformed destruction operator. The detailed form will have a SQRT(E/E') factor, where E' is the transformed energy -- change of measure. This whole subject is extensively discussed in Weinberg's book.

Regards,
Reilly Atkinson
 

Similar threads

  • Quantum Physics
Replies
4
Views
954
Replies
2
Views
711
  • Quantum Physics
Replies
3
Views
954
Replies
21
Views
2K
  • Quantum Physics
Replies
9
Views
1K
Replies
2
Views
378
  • Quantum Physics
Replies
13
Views
1K
  • Quantum Physics
Replies
4
Views
770
  • Quantum Physics
Replies
15
Views
2K
  • Quantum Physics
Replies
4
Views
1K
Back
Top